Proving F = grad(f) with 3 Variables

  • Thread starter Thread starter jwxie
  • Start date Start date
Click For Summary
For a vector field F in three dimensions to be the gradient of a scalar field f, it must satisfy the condition of having zero curl, expressed as ∇×F = 0. This results in three equations relating the partial derivatives of the components of F. Specifically, it requires that the differences between the partial derivatives of the components with respect to different variables equal zero. The proof parallels the two-variable case, relying on the concept that a gradient vector field has a zero integral around closed loops, which allows for the construction of a potential function f. Understanding these conditions is crucial for proving F = grad(f) in three variables.
jwxie
Messages
278
Reaction score
0

Homework Statement



For two variables, F = grad(f) if only if partial P / partial y is equal to partial Q / partial x
where, P and Q represent the x, y function, P(x,y) and Q(x,y)

For example:
F(x,y) = P(x,y) + Q(x,y)

Now the question is, for three variables, I have P + Q + R.
Is proving partial P / partial y = partial Q / partial x enough to say that F = grad(f)? If not, what is the approach to prove it for a 3 variables?

Thank you
 
Physics news on Phys.org
The condition for a vector field in three-space to be the gradient of some scalar field is a generalization of the condition in the plane: the vector field must have zero curl, \nabla\times\mathbf{F} = 0. This works out to three equations, not one, between the partial derivatives of the components of \mathbf{F}: \frac{\partial F_x}{\partial y} - \frac{\partial F_y}{\partial x} = \frac{\partial F_x}{\partial z} - \frac{\partial F_z}{\partial x} = \frac{\partial F_z}{\partial y} - \frac{\partial F_y}{\partial z} = 0.

The proof of this fact uses the same argument as in the plane case (a gradient vector field is one whose integral around a closed loop is always zero, so you can construct a potential function f with \mathbf{F} = \nabla f by integration) together with Stokes's theorem.
 
Thank you very much!
 
Question: A clock's minute hand has length 4 and its hour hand has length 3. What is the distance between the tips at the moment when it is increasing most rapidly?(Putnam Exam Question) Answer: Making assumption that both the hands moves at constant angular velocities, the answer is ## \sqrt{7} .## But don't you think this assumption is somewhat doubtful and wrong?

Similar threads

Replies
5
Views
2K
  • · Replies 4 ·
Replies
4
Views
2K
  • · Replies 4 ·
Replies
4
Views
1K
  • · Replies 1 ·
Replies
1
Views
1K
  • · Replies 18 ·
Replies
18
Views
3K
  • · Replies 8 ·
Replies
8
Views
2K
  • · Replies 9 ·
Replies
9
Views
2K
  • · Replies 7 ·
Replies
7
Views
2K
  • · Replies 2 ·
Replies
2
Views
2K
Replies
1
Views
2K